Monday, April 11, 2011

Cardiovascular Pathophysiology MCQ test

  1. A one-month-old infant is taken to the pediatrician because of dyspnea, difficulty in feeding, and poor weight gain. Physical examination reveals tachypnea, a weak femoral pulse compared to the radial pulse, and a 30 mm Hg difference in systolic pressure between the upper and lower extremities. Which of the following is the most likely diagnosis?


    1. Aortic coarctation

    1. Aortic regurgitation

    1. Cardiac tamponade

    1. Heart failure

    1. Hypovolemia


  1. Uncontrollable bleeding from the umbilical stump of a neonate specifically suggests deficiency of which of the following coagulation factors?


    1. Factor VIII

    1. Factor IX

    1. Factor XII

    1. Factor XIII

    1. Von Willebrand's factor


  1. A high school basketball player passes out in the middle of a game. He is rushed to the emergency room, where he regains consciousness. He claims that just before he fainted, he had difficulty breathing and experienced palpitations. On physical exam, he has a bifid apical impulse and a coarse systolic murmur at the left sternal border. The echocardiogram reveals ventricular hypertrophy with asymmetric septal thickening. Which of the following would increase the intensity of his heart murmur?


    1. Elevating his legs

    1. Increasing sympathetic tone

    1. Performing the Valsalva maneuver

    1. Squatting

  1. A 58-year-old man with fainting spells and exercise intolerance is found to have a bicuspid aortic valve with marked aortic stenosis. Which of the following physical findings would be prominent in this patient?



    1. Diastolic murmur

    1. Heave at left parasternal border

    1. Loud S2 heart sound

    1. Loud S3 heart sound

    1. Weak peripheral pulse

  1. A 1-year-old child with frequent pulmonary infections has signs of mild congestive heart failure. On physical examination, he is found to have a harsh systolic murmur with no diastolic murmur. Cardiac catheterization shows increased oxygen saturation in the right ventricle. Two-dimensional echocardiography would most likely reveal


    1. aortic insufficiency

    1. mitral stenosis

    1. a patent ductus arteriosus

    1. a patent foramen ovale

    1. a ventricular septal defect


  1. A 47-year-old male enters the hospital emergency room after severing a major artery during a farm accident. It is estimated that the patient lost about 800 mL of blood. His blood pressure is 95/65 mm Hg. A decrease in which of the following would be expected in response to hemorrhage in this man?

    1. Heart rate

    1. Plasma renin activity

    1. Sympathetic nerve activity

    1. Total peripheral resistance

    1. Vagal nerve activity

  1. ear-old woman complains to her physician that she is chronically tired. She has lost several pounds in the past few months without a change in her diet. Blood tests indicate she has severe anemia (Hb < 7 g/dL). Further testing shows the presence of blood products in her stool and a large malignant tumor in her ascending colon.
Which of the following is likely to be decreased in this woman?


    1. Arterial O2 content

    1. Arterial O2 saturation

    1. Arterial PO2

    1. Cardiac output

    1. Heart rate

    1. Stroke volume


  1. An increase in which of the following best explains the mechanism by which the cardiac output increases in severe anemia?

    1. Arteriolar diameter

    1. Blood viscosity

    1. Peripheral vascular resistance

    1. Splanchnic blood flow

    1. Tissue oxygen tension


  1. A 57-year-old female with renal insufficiency has been on dialysis for thirteen years, but has failed to make her last two appointments. She presents to the emergency room in obvious distress with a blood pressure of 85/40 and jugular venous distension. Cardiac auscultation reveals no murmurs, thrills, or heaves. Her heart rate is rapid, at 108 beats/min, and the peripheral pulses are thready. Pulsus paradoxus is present, but Kussmaul's sign is absent. Echocardiography reveals the presence of a small heart. Which of the following is the most likely diagnosis?


    1. Cardiac tamponade

    1. Constrictive pericarditis

    1. Congestive heart failure

    1. Myocardial infarct

    1. Restrictive cardiomyopathy

  1. A 40-year-old man presents to his physician with complaints of chest pain. The pain is paroxysmal, substernal, and occurs while at rest. An electrocardiogram performed in the doctor's office shows ST segment elevations.

Which of the following is the most likely cause of the man's pain?


    1. Heartburn

    1. Hypertension

    1. Severe atherosclerotic narrowing of coronary arteries

    1. Thromboembolism of coronary arteries

    1. Vasospasm of atherosclerotic vessels
Answers:
1. A 2. D 3. C 4. E 5. E 6. E 7. A 8. A 9. A 10. E